Digital SAT Practice Exam Questions

Draft Questions - level 1

words in context

1,4,7,10 – 2,3,5,9

  1. The Grand Canyon is one of the most visited national parks in the United States. The vast, rugged landscape attracts millions of visitors each year who come to ______ its breathtaking views and geological significance.

   – A. preserve

   – B. admire

   – C. obscure

   – D. diminish

  1. In her autobiography, the author recounts how she ______ the challenges of her early life by focusing on her education and artistic pursuits.

   – A. navigated

   – B. complicated

   – C. overlooked

   – D. dismissed

  1. The new urban development project aims to ______ the city’s infrastructure by improving public transportation and increasing green spaces.

   – A. neglect

   – B. degrade

   – C. revitalize

   – D. impair

  1. The scientist’s groundbreaking research has significantly ______ our understanding of climate change and its impacts on global ecosystems.

   – A. obscured

   – B. confused

   – C. expanded

   – D. distorted

  1. The playwright’s use of humor and irony helps to ______ the serious themes addressed in the play, making them more accessible to the audience.

   – A. amplify

   – B. diminish

   – C. clarify

   – D. complicate

  1. As the company expanded its operations globally, it became necessary to ______ the diverse cultural norms and practices of the new markets.

   – A. disregard

   – B. embrace

   – C. dismiss

   – D. challenge

  1. The ancient ruins were carefully ______ by archaeologists who wanted to preserve their historical value while making them accessible to the public.

   – A. abandoned

   – B. disregarded

   – C. restored

   – D. obliterated

  1. The documentary filmmaker was praised for his ability to ______ the complex social issues in a way that was both informative and engaging.

   – A. simplify

   – B. sensationalize

   – C. ignore

   – D. misrepresent

  1. The author’s latest novel is praised for its ability to ______ the reader’s emotions, drawing them into the protagonist’s journey with empathy and understanding.

   – A. alienate

   – B. manipulate

   – C. engage

   – D. confuse

  1. The conservationist’s efforts to ______ the endangered species were met with widespread support from the global community.

   – A. ignore

   – B. exploit

   – C. protect

   – D. harm

Text Structure & Purpose

3, 7, and 9 i

  1.  

The Great Wall of China, originally built to protect the Chinese states from invasions, has become one of the most famous landmarks in the world. It spans over 13,000 miles and includes various types of walls, towers, and fortifications built by different dynasties over many centuries. Today, the Great Wall is not only a symbol of Chinese strength and perseverance but also a major tourist attraction, drawing millions of visitors each year.

Which choice best states the main purpose of the text? 

  1. A) To explain the original purpose of the Great Wall of China.
  2. B) To highlight the Great Wall of China as a tourist attraction.
  3. C) To describe the different structures that make up the Great Wall of China.
  4. D) To discuss the historical significance of the Great Wall of China.
  1.  

During the 19th century, the advent of the steam engine transformed transportation and industry. Steam engines allowed for faster and more reliable transportation of goods and people, contributing to the expansion of markets and the growth of cities. This technological innovation also led to significant social changes, including the rise of the working class and the development of new forms of labor.

Which choice best describes the function of the text as a whole? 

  1. A) It explains the technological advancements of the 19th century.
  2. B) It discusses the social impact of the steam engine.
  3. C) It highlights the role of the steam engine in the industrial revolution.
  4. D) It details the economic changes brought about by the steam engine.
  1.  

In recent years, scientists have discovered that certain species of birds are capable of using tools to obtain food. For example, crows have been observed using sticks to extract insects from tree bark. This behavior challenges the traditional view that tool use is a uniquely human trait and suggests that the cognitive abilities of birds may be more complex than previously thought.

Which choice best states the main purpose of the text? 

  1. A) To describe the tool-using behavior of certain bird species.
  2. B) To challenge the idea that tool use is unique to humans.
  3. C) To provide examples of cognitive abilities in birds.
  4. D) To discuss the evolution of tool use in animals.
  1.  

The text is from “The War of the Worlds,” a science fiction novel by H.G. Wells. 

Across the Gulf of Space, the Martians moved to create machines of war to conquer Earth. It is said that the Martians envied Earth’s lush vegetation, fresh water, and mild climate, for their own planet was dying. Despite their advanced technology, they lacked the resources to sustain their race and thus sought to dominate our world.

Which choice best describes the function of the underlined sentence? 

  1. A) It highlights the Martians’ motivation for invading Earth.
  2. B) It contrasts the Martians’ world with Earth.
  3. C) It foreshadows the conflict between Earth and Mars.
  4. D) It explains the advanced technology of the Martians.
  1.  

Studies have shown that exercise can significantly improve mental health by reducing symptoms of depression and anxiety. Regular physical activity increases the production of endorphins, which are chemicals in the brain that help to improve mood and reduce stress. Additionally, exercise promotes better sleep and increases self-esteem, both of which contribute to overall well-being.

Which choice best states the main purpose of the text? 

  1. A) To explain the chemical process that occurs in the brain during exercise.
  2. B) To describe the various benefits of exercise on mental health.
  3. C) To encourage readers to engage in regular physical activity.
  4. D) To discuss the relationship between exercise and self-esteem.
  1.  

The following text is from the novel “Pride and Prejudice” by Jane Austen. 

Elizabeth Bennet had often heard her mother say that she would be married before any of her sisters. However, as she observed the manners of the gentlemen in her acquaintance, she became increasingly determined to marry only for love, not for convenience. This decision set her apart from many women of her time, who often married out of necessity rather than affection.

Which choice best describes the function of the underlined sentence? 

  1. A) It highlights Elizabeth’s resolve to marry for love.
  2. B) It contrasts Elizabeth’s views with those of her mother.
  3. C) It foreshadows the challenges Elizabeth will face in finding a suitable husband.
  4. D) It introduces the central conflict in Elizabeth’s life.
  1.  

The following text is from a speech by Martin Luther King Jr. 

We have come to our nation’s capital to cash a check. When the architects of our republic wrote the magnificent words of the Constitution and the Declaration of Independence, they were signing a promissory note to which every American was to fall heir. This note was a promise that all men would be guaranteed the unalienable rights of life, liberty, and the pursuit of happiness. It is obvious today that America has defaulted on this promissory note insofar as her citizens of color are concerned. Instead of honoring this sacred obligation, America has given the Negro people a bad check, a check that has come back marked “insufficient funds.”

Which choice best states the main purpose of the text? 

  1. A) To criticize the government’s failure to uphold civil rights.
  2. B) To demand equality and justice for all Americans.
  3. C) To highlight the broken promises made to African Americans.
  4. D) To call for immediate action to address racial inequality.
  1.  

In the early 20th century, the United States saw a surge in the construction of skyscrapers. These towering structures became symbols of modernity and economic power, representing the rapid growth of American cities. Architects and engineers competed to design taller and more innovative buildings, leading to the development of iconic landmarks such as the Empire State Building and the Chrysler Building.

Which choice best describes the function of the text as a whole? 

  1. A) It explains the architectural innovations of the early 20th century.
  2. B) It discusses the significance of skyscrapers in American culture.
  3. C) It highlights the competition among architects to build the tallest buildings.
  4. D) It provides a historical overview of skyscraper construction in the United States.
  1.  

The following text is from a scientific article about the effects of climate change on polar bear populations. 

As the Arctic ice continues to melt due to rising global temperatures, polar bears are finding it increasingly difficult to hunt for their primary prey, seals. With less ice available, polar bears must swim longer distances to find food, leading to exhaustion and decreased survival rates. Researchers predict that if current trends continue, polar bears could face severe population declines in the coming decades.

Which choice best states the main purpose of the text? 

  1. A) To explain the impact of climate change on polar bear habitats.
  2. B) To discuss the challenges polar bears face in finding food.
  3. C) To highlight the potential consequences of declining polar bear populations.
  4. D) To advocate for increased efforts to combat climate change.
  1.  

The following text is from a historical account of the Industrial Revolution. 

The invention of the spinning jenny by James Hargreaves in 1764 revolutionized the textile industry by allowing workers to spin multiple threads at once. This invention increased production speed and efficiency, leading to the growth of textile factories and the expansion of the industry. However, the rapid industrialization also had negative consequences, such as poor working conditions and the exploitation of labor, particularly among women and children.

Which choice best describes the function of the underlined sentence? 

  1. A) It introduces the invention that sparked the Industrial Revolution.
  2. B) It explains the significance of the spinning jenny in the textile industry.
  3. C) It contrasts the positive and negative effects of industrialization.
  4. D) It highlights the impact of the spinning jenny on labor practices.

Cross Text connections

1,2 – 4,3 spare 6,7

  1. Text 1

The history of astronomy is a story of increasing precision. Ancient astronomers used basic tools to chart the stars and predict celestial events. In the 17th century, the invention of the telescope marked a significant leap in accuracy, allowing scientists to observe distant objects in detail. Today, space telescopes like the Hubble provide unprecedented clarity, pushing the boundaries of our understanding of the universe.

   Text 2 

While the history of astronomy has certainly been shaped by technological advancements, it is also a tale of changing ideas. The transition from a geocentric (Earth-centered) model of the solar system to a heliocentric (Sun-centered) one in the 16th century revolutionized our understanding of the cosmos. This shift was not just a technological breakthrough but a fundamental change in how humanity viewed its place in the universe.

   Which choice best describes how the author of Text 2 expands on the ideas in Text 1? 

   – A. The author of Text 2 focuses on the role of technological advancements in the history of astronomy, while the author of Text 1 emphasizes changing ideas. 

   – B. The author of Text 2 highlights the importance of conceptual shifts in astronomy, adding depth to the technological progress discussed in Text 1. 

   – C. The author of Text 2 disagrees with the notion that technological advancements have been the most important aspect of astronomical history. 

   – D. The author of Text 2 argues that the transition from geocentric to heliocentric models was primarily a result of technological innovation.

  1. Text 1

Climate change has led to a growing interest in renewable energy sources. Solar power, in particular, has gained popularity due to its potential to provide clean, sustainable energy. Innovations in solar technology have made it more efficient and affordable, leading to its widespread adoption in both residential and commercial settings.

   Text 2 

While solar power offers many environmental benefits, it is not without challenges. The production of solar panels involves the use of hazardous materials, and the panels themselves have a limited lifespan, after which they must be disposed of. Additionally, solar power generation is highly dependent on weather conditions, which can be unpredictable.

   Which choice best describes the relationship between the two texts? 

   – A. Text 2 introduces concerns that complicate the positive view of solar power presented in Text 1. 

   – B. Text 2 contradicts the claim in Text 1 that solar power is a sustainable energy source. 

   – C. Text 2 provides additional evidence to support the benefits of solar power discussed in Text 1. 

   – D. Text 2 shifts the focus from the environmental benefits of solar power to its economic implications.

  1. Text 1

The use of antibiotics in livestock has become a controversial topic. Proponents argue that antibiotics are necessary to prevent disease and ensure the health of animals, which in turn supports the global food supply. However, critics warn that the overuse of antibiotics in agriculture contributes to the development of antibiotic-resistant bacteria, which pose a serious threat to public health.

   Text 2 

In response to concerns about antibiotic resistance, some countries have implemented stricter regulations on the use of antibiotics in livestock. These measures include banning the use of certain antibiotics and promoting alternative practices, such as improved hygiene and the use of vaccines. While these efforts have been met with some resistance from the agricultural industry, they are seen as crucial steps in combating the spread of antibiotic resistance.

   Which choice best describes how the author of Text 2 responds to the concerns raised in Text 1? 

   – A. The author of Text 2 dismisses the concerns about antibiotic resistance as overstated. 

   – B. The author of Text 2 provides solutions to the problems associated with antibiotic use in livestock discussed in Text 1. 

   – C. The author of Text 2 argues that the benefits of antibiotic use in livestock outweigh the risks. 

   – D. The author of Text 2 questions the effectiveness of the regulations on antibiotic use in livestock.

  1. Text 1

The study of human evolution has long been dominated by the search for fossil evidence. These fossils provide crucial insights into the physical characteristics and behaviors of early human ancestors. Recent discoveries have shed light on previously unknown species, offering new perspectives on the evolutionary tree.

   Text 2 

In addition to fossils, genetic research has become an increasingly important tool in the study of human evolution. By analyzing the DNA of modern humans and comparing it to that of other species, scientists have been able to trace evolutionary lineages and identify genetic mutations that played key roles in human development. This genetic approach has complemented and sometimes even challenged traditional fossil-based theories.

   Which choice best describes how the two texts are related? 

   – A. Text 2 expands on the methods discussed in Text 1 by introducing genetic research as a complementary tool. 

   – B. Text 2 challenges the validity of fossil evidence as the primary source of information about human evolution. 

   – C. Text 2 argues that genetic research is more important than fossil evidence in understanding human evolution. 

   – D. Text 2 suggests that fossil evidence is outdated and should be replaced by genetic research.

  1. Text 1

Urban farming has gained attention as a way to address food insecurity in cities. By growing food locally, urban farms can reduce the need for transportation and provide fresh produce to underserved communities. Additionally, urban farming can transform vacant lots into productive green spaces, contributing to community revitalization.

   Text 2 

While urban farming has many potential benefits, it also faces significant challenges. Urban farms often struggle with limited space, poor soil quality, and a lack of resources. Moreover, the scale of urban farming is generally too small to make a substantial impact on overall food production in cities. Despite these obstacles, proponents argue that urban farming is a valuable tool for raising awareness about sustainable agriculture.

   Which choice best describes the relationship between the two texts? 

   – A. Text 2 provides a more realistic view of the limitations of urban farming compared to the optimistic perspective in Text 1. 

   – B. Text 2 refutes the claim in Text 1 that urban farming can contribute to community revitalization. 

   – C. Text 2 argues that the benefits of urban farming are exaggerated in Text 1. 

   – D. Text 2 suggests that urban farming should be abandoned in favor of more effective solutions.

  1. Text 1

Classical music, often considered a form of “high art,” has a long history of being associated with the elite. However, in recent years, there has been a push to make classical music more accessible to the general public. Initiatives such as free outdoor concerts and educational programs in schools aim to broaden the audience for classical music and break down the barriers that have traditionally kept it exclusive.

   Text 2 

Despite efforts to make classical music more inclusive, challenges remain. The genre is still often perceived as outdated and irrelevant to contemporary life. Additionally, the cost of attending live performances can be prohibitive for many people. Critics argue that for classical music to truly become more accessible, it must evolve to reflect modern tastes and concerns, perhaps by incorporating elements of popular music or addressing current social issues.

   Which choice best describes how the author of Text 2 views the initiatives described in Text 1? 

   – A. The author of Text 2 believes that the initiatives are unlikely to succeed unless classical music undergoes significant changes. 

   – B. The author of Text 2 is optimistic that the initiatives will make classical music more accessible. 

   – C. The author of Text 2 dismisses the initiatives as ineffective in broadening the audience for classical music. 

   – D. The author of Text 2 argues that the initiatives have already had a significant impact on making classical music more inclusive.

  1. Text 1

The rise of online education has been hailed as a revolution in learning. Online courses offer flexibility and accessibility, allowing students to learn at their own pace and from anywhere in the world. Proponents argue that online education can democratize access to knowledge and provide opportunities for lifelong learning.

   Text 2 

While online education has many advantages, it also presents challenges. One major concern is the lack of face-to-face interaction, which can lead to feelings of isolation and a lack of engagement among students. Additionally, not all students have access to the necessary technology or a stable internet connection, which can exacerbate educational inequalities. Critics argue that online education should be seen as a complement to, rather than a replacement for, traditional classroom learning.

   Which choice best describes how the two texts relate to each other? 

   – A. Text 2 introduces potential drawbacks to the optimistic view of online education presented in Text 1. 

   – B. Text 2 argues that online education is superior to traditional classroom learning, contradicting Text 1. 

   – C. Text 2 supports the claim in Text 1 that online education can democratize access to knowledge. 

   – D. Text 2 suggests that online education is more effective for lifelong learning than for formal education.

  1. Text 1

Ecotourism is often promoted as a way to support conservation efforts while providing economic benefits to local communities. By encouraging tourists to visit natural areas, ecotourism can generate funds for the protection of endangered species and habitats. It also offers local communities an alternative to activities that harm the environment, such as logging or mining.

   Text 2

While ecotourism has the potential to support conservation, it can also have negative effects. The influx of tourists can put pressure on fragile ecosystems, leading to habitat degradation and disturbance to wildlife. Moreover, the economic benefits of ecotourism are not always evenly distributed, with profits often going to outside operators rather than local communities. Critics argue that without careful management, ecotourism can do more harm than good.

   Which choice best describes how the two texts are related? 

   – A. Text 2 highlights the potential downsides of ecotourism, complicating the positive view presented in Text 1. 

   – B. Text 2 contradicts the claim in Text 1 that ecotourism provides economic benefits to local communities. 

   – C. Text 2 supports the idea in Text 1 that ecotourism can help protect endangered species and habitats. 

   – D. Text 2 argues that ecotourism should be replaced with more sustainable forms of tourism.

  1. Text 1

The development of electric vehicles (EVs) has been a major focus of the automotive industry in recent years. EVs offer a cleaner alternative to traditional gasoline-powered cars, reducing greenhouse gas emissions and dependence on fossil fuels. Advances in battery technology have also improved the range and performance of EVs, making them more practical for everyday use.

   Text 2 

Despite the environmental benefits of EVs, challenges remain. The production of batteries for EVs involves the extraction of rare metals, which can have significant environmental and social impacts. Additionally, the infrastructure for charging EVs is still limited in many areas, making it difficult for drivers to rely solely on electric power. Critics argue that while EVs are a step in the right direction, they are not a complete solution to the environmental challenges posed by transportation.

   Which choice best describes the relationship between the two texts? 

   – A. Text 2 acknowledges the benefits of EVs but introduces challenges that complicate their adoption. 

   – B. Text 2 argues that EVs are not as environmentally friendly as Text 1 suggests. 

   – C. Text 2 supports the claim in Text 1 that advances in battery technology have improved the practicality of EVs. 

   – D. Text 2 suggests that alternative forms of transportation are more environmentally sustainable than EVs.

  1. Text 1

Renewable energy sources, such as wind and solar power, are often cited as key solutions to reducing greenhouse gas emissions. These sources are abundant and generate energy without producing harmful pollutants. As the technology for harnessing renewable energy improves, it is becoming increasingly cost-effective, leading to a rise in adoption worldwide.

    Text 2 

While renewable energy sources offer significant environmental benefits, they also have limitations. Wind and solar power are intermittent, meaning they do not produce energy consistently. This variability can pose challenges for grid management, requiring backup power sources or energy storage solutions. Additionally, the infrastructure for renewable energy is still developing, and transitioning away from fossil fuels will take time.

    Which choice best describes how the author of Text 2 views the potential of renewable energy sources? 

    – A. The author of Text 2 sees renewable energy as a promising but imperfect solution to reducing greenhouse gas emissions. 

    – B. The author of Text 2 believes that renewable energy sources are insufficient to replace fossil fuels entirely. 

    – C. The author of Text 2 argues that the environmental benefits of renewable energy are overstated. 

    – D. The author of Text 2 suggests that renewable energy will eventually become the dominant source of power worldwide.

Central Idea

1,7,4 – 3,2,6

Question 1:

The Australian government has implemented a conservation program to protect the Great Barrier Reef. The program includes measures such as limiting coastal development, improving water quality, and controlling fishing activities. These efforts are intended to preserve the reef’s biodiversity and maintain its role as a major attraction for tourists.

Which choice best states the main idea of the text?

– A) Coastal development is the biggest threat to the Great Barrier Reef.

– B) The Australian government is taking steps to protect the Great Barrier Reef.

– C) Tourism is the primary reason for the conservation of the Great Barrier Reef.

– D) Fishing activities are being restricted to preserve the Great Barrier Reef.

Question 2:

In recent years, the concept of “slow fashion” has gained popularity as consumers become more aware of the environmental impact of their clothing choices. Slow fashion emphasizes the importance of buying high-quality, durable garments and supporting ethical brands that prioritize sustainability over mass production.

Which choice best states the main idea of the text?

– A) Consumers are increasingly choosing slow fashion over fast fashion.

– B) Slow fashion promotes sustainability and ethical clothing production.

– C) Mass production is less environmentally harmful than slow fashion.

– D) Slow fashion is a recent trend with minimal impact on consumer behavior.

Question 3:

Researchers have discovered that some plants release chemical signals when they are under attack by herbivores. These chemicals can attract predators of the herbivores, thus helping to protect the plants. This discovery highlights the complex interactions between plants and other organisms in their ecosystems.

Which choice best states the main idea of the text?

– A) Plants can defend themselves by releasing chemicals that attract predators of their attackers.

– B) Herbivores are the primary threat to plants in various ecosystems.

– C) The release of chemicals by plants is a rare phenomenon in nature.

– D) Researchers are studying how plants interact with herbivores in different ecosystems.

Question 4:

The study of ancient civilizations has revealed that many of them developed advanced agricultural practices. These practices included irrigation systems, crop rotation, and the domestication of plants and animals. Such innovations allowed these civilizations to sustain large populations and thrive for centuries.

Which choice best states the main idea of the text?

– A) Ancient civilizations were successful because of their advanced agricultural practices.

– B) Crop rotation was the most important agricultural innovation in ancient civilizations.

– C) The domestication of plants and animals was crucial to the development of ancient civilizations.

– D) Advanced agricultural practices were limited to a few ancient civilizations.

Question 5:

Modern technology has significantly changed the way we communicate. The rise of social media platforms, instant messaging, and video calls has made it easier for people to stay connected, regardless of their physical location. However, some argue that these changes have also led to a decrease in face-to-face interactions and a loss of personal connection.

Which choice best states the main idea of the text?

– A) Social media has improved communication but reduced face-to-face interactions.

– B) Modern technology has revolutionized communication by making it more convenient.

– C) Video calls are the most effective way to maintain personal connections.

– D) Instant messaging has replaced other forms of communication in modern society.

Question 6:

Urban planners are increasingly recognizing the importance of green spaces in cities. Parks, gardens, and green rooftops provide residents with areas for recreation and relaxation, improve air quality, and contribute to the overall well-being of the urban population. As a result, more cities are incorporating green spaces into their development plans.

Which choice best states the main idea of the text?

– A) Green spaces are essential for improving air quality in cities.

– B) Urban planners are including more green spaces in city development.

– C) Residents benefit from having access to parks and gardens in cities.

– D) Green rooftops are a new trend in urban development.

Question 7:

Scientists have developed a new method for generating electricity using the movement of ocean waves. This technology harnesses the kinetic energy of waves and converts it into electrical energy, which can then be used to power homes and businesses. The method is seen as a promising source of renewable energy, especially for coastal regions.

Which choice best states the main idea of the text?

– A) Ocean wave energy is a promising new source of renewable energy.

– B) Coastal regions are the primary beneficiaries of ocean wave energy.

– C) Scientists are working on improving the efficiency of wave energy technology.

– D) The movement of ocean waves can generate a significant amount of electricity.

Question 8:

Public libraries have long been a cornerstone of communities, providing free access to books, information, and educational resources. In recent years, many libraries have expanded their offerings to include digital resources, technology workshops, and community events, further solidifying their role as important centers of learning and engagement.

Which choice best states the main idea of the text?

– A) Public libraries are adapting to the digital age by offering new services.

– B) Public libraries remain essential community resources in the digital age.

– C) Technology workshops are the most popular programs at public libraries.

– D) Community events at libraries are becoming more common than book lending.

Question 9:

The discovery of antibiotics in the early 20th century revolutionized medicine, making it possible to treat bacterial infections that were once deadly. However, the overuse and misuse of antibiotics have led to the rise of antibiotic-resistant bacteria, posing a serious challenge to public health today.

Which choice best states the main idea of the text?

– A) Antibiotics have been overused, leading to antibiotic-resistant bacteria.

– B) The discovery of antibiotics was a major milestone in medical history.

– C) Antibiotic-resistant bacteria are a growing public health concern.

– D) The misuse of antibiotics has diminished their effectiveness.

Question 10:

The concept of a circular economy is gaining traction as a way to reduce waste and promote sustainability. Unlike the traditional linear economy, where products are made, used, and then discarded, a circular economy focuses on designing products that can be reused, repaired, or recycled, thus minimizing environmental impact.

Which choice best states the main idea of the text?

– A) The circular economy is an emerging model for reducing waste and promoting sustainability.

– B) Reusing and repairing products are key components of the circular economy.

– C) The linear economy is being replaced by the circular economy in many industries.

– D) A circular economy reduces waste by focusing on product design.

Data Evidence

Textual evidence

1,4, – 2,3

  1. The Milky Way galaxy is composed of millions of stars in a relatively flat structure containing a thin disk and a thick disk. Based on computer simulations and analysis of data on the brightness, position, and chemical composition of about 250,000 stars in the thick disk (collected from two telescopes, one in China and one orbiting in space), astrophysicists Maosheng Xiang and Hans-Walter Rix claim that the thick disk of the Milky Way formed in two distinct phases rather than a single one.

   – Which finding, if true, would most directly support the researchers’ claim?

     – A) There’s an age difference of about 2 billion years between certain stars in the thick disk.

     – B) The stars in the Milky Way tend to have very similar chemical compositions.

     – C) The thin disk contains about twice as many stars that can be seen from Earth as the thick disk does.

     – D) The telescopes used by the researchers have detected stars of similar ages in galaxies other than the Milky Way.

  1. Roasted green chiles are a popular ingredient in Southwestern cuisine, but the traditional roasting method of burning propane is not environmentally friendly. To see if solar power could provide a better alternative, engineer Kenneth Armijo and his team roasted batches of green chiles using between 38 and 42 heliostats, which are devices that concentrate sunlight. The team was successful in reaching the same roasting temperature used in traditional propane roasting, but they found that propane yielded faster results. While the fastest solar-roasted green chiles took six minutes, batches using propane took only four. Armijo hypothesizes that they can reduce the roasting time for solar-roasted green chiles by using more heliostats.

   – Which finding, if true, would most directly support Armijo’s hypothesis?

     – A) Attempts to roast green chiles using 50 heliostats yield results in fewer than six minutes.

     – B) Green chile connoisseurs prefer the flavor of solar-roasted green chiles over the flavor of propane roasted green chiles.

     – C) The skins of solar-roasted green chiles are easier to peel than the skins of propane-roasted green chiles.

     – D) The temperature inside the roasting drum is distributed more evenly when roasting green chiles with solar power than with propane.

  1. A 2019 study published in Nature Ecology & Evolution looked at data from more than 1,500 species of birds, mammals, and fish in order to determine whether their rate of evolutionary change was linked to species survival. The study found that while some species did evolve faster than others, there was no clear relationship between evolutionary rate and survival. Instead, the study suggests that previous species success predicts species survival much more accurately than speed of adaptation does.

   – Which finding, if true, would most directly support the underlined claim?

     – A) Endangered and extinct species displayed a significantly slower evolutionary rate than other species.

     – B) Evolutionary rate only impacted species survival when adaptations were linked to environmental pressures.

     – C) Successful species tended to remain more successful than more quickly-evolving competitors.

     – D) Invasive species that overtook competitors tended to exhibit a higher evolutionary rate than other species.

  1. Some residents in a neighborhood in Atlanta recently founded a community garden inside a local park. The residents agreed to volunteer to take care of the garden together. Students at a local high school surveyed some of the volunteers as part of a project to understand the impact of the new garden. The students concluded that the new garden benefited the community overall by fostering connections and relationships between the volunteers and other residents of the neighborhood who weren’t volunteering at the garden.

   – Which quotation from a survey respondent would best illustrate the students’ conclusion?

     – A) “I love getting the opportunity to be outside and around nature, especially on days when the weather is nice.”

     – B) “We’re lucky to have a few expert gardeners living in the neighborhood. Some volunteers and I have gone to them a few times with questions, and they’ve been eager to help us and to learn more about the project.”

     – C) “My favorite thing about the garden is the feeling of pride I get when I walk by each day. As I see the plants growing, I feel good knowing I had a small part in creating this beautiful space in the neighborhood.”

     – D) “Our first challenge was deciding what plants would be most suitable to the climate and soil here in Atlanta. We needed plants that could survive the hot and humid summers.”

  1. Researchers are studying a specific species of desert plants that have developed a unique mechanism to minimize water loss through their leaves. They found that the plants close their stomata (tiny openings on the leaf surface) during the hottest part of the day to reduce water loss.

   – Which of the following, if true, would most directly support the researchers’ conclusion?

     – A) Other desert plants with similar leaf structures also close their stomata during the hottest part of the day.

     – B) The plants are more likely to survive in areas with higher average temperatures.

     – C) The stomata in these plants open widely during the cooler parts of the day to allow gas exchange.

     – D) The plants also have deep root systems that help them access water from underground sources.

  1. A new study shows that ocean temperatures have increased by 1.5°C over the past century. The study suggests that this temperature rise has caused a significant decline in the population of certain fish species that are sensitive to temperature changes.

   – Which piece of evidence would most directly support the study’s suggestion?

     – A) The decline in fish populations coincided with the period of temperature rise.

     – B) Fish populations in areas where the temperature did not rise remained stable.

     – C) Some fish species have migrated to cooler waters as a result of the temperature rise.

     – D) Fish species with a higher tolerance for temperature changes have not experienced population declines.

  1. A sociologist conducted a study on the impact of remote work on employee productivity. The study found that employees who worked remotely had higher productivity levels than those who worked in an office environment.

   – Which finding, if true, would most directly support the study’s conclusion?

     – A) Employees who worked remotely reported lower stress levels than those who worked in an office.

     – B) Remote workers had more flexible work hours, allowing them to work during their most productive times.

     – C) Employees working from home had fewer distractions compared to those working in an office.

     – D) Office workers had access to better resources and technology than remote workers.

  1. In a recent survey, customers of a popular coffee chain were asked about their preferences for different types of coffee. The survey found that most customers preferred a dark roast over a light roast.

   – Which finding, if true, would most directly support the survey’s results?

     – A) Sales data from the coffee chain shows that dark roast coffees are purchased more frequently than light roast coffees.

     – B) Customers who prefer dark roast coffee tend to visit the coffee chain more often than those who prefer light roast.

     – C) The coffee chain has more varieties of dark roast coffee than light roast coffee.

     – D) Customer reviews indicate that dark roast coffee has a richer flavor than light roast coffee.

  1. A group of scientists is studying the effects of deforestation on the local wildlife population in a tropical rainforest. The scientists hypothesize that deforestation is leading to a decline in the population of certain bird species.

   – Which piece of evidence would most directly support the scientists’ hypothesis?

     – A) Bird species that rely on specific tree species for nesting have seen the sharpest population declines.

     – B) The overall bird population in the area has declined over the past decade.

     – C) Some bird species have adapted to the changes in their habitat caused by deforestation.

     – D) The rate of deforestation in the area has increased significantly in recent years.

  1. A historian is researching the impact of a major economic recession on small businesses in a particular region. The historian found that many small businesses failed during the recession, but some managed to survive.

    – Which piece of evidence would most directly support the historian’s findings?

      – A) Small businesses that diversified their products or services were more likely to survive the recession.

      – B) Many small businesses that failed during the recession were already struggling financially before the recession began.

      – C) Government aid programs were available to small businesses during the recession.

      – D) The region’s economy took several years to recover from the recession.

Inferences

2,4,5 – 1,3,6

  1. Text 1

Many people believe that urban gardening is a way to reconnect with nature and improve mental well-being. By growing plants in city environments, people can experience the calming effects of nature without leaving their homes. However, urban gardening also presents challenges, such as limited space and soil quality. Consequently, while urban gardening can offer significant personal benefits, it also requires creative solutions to address these challenges. 

Which choice best reflects an inference that can be drawn from the text? 

  1. A) Urban gardening provides people with an opportunity to escape their busy city lives.
  2. B) The personal benefits of urban gardening often outweigh the practical challenges involved.
  3. C) Many city dwellers are dissatisfied with their current gardening options.
  4. D) Urban gardening is a temporary trend that will soon fade away.
  1. Text 2

In recent years, there has been growing interest in sustainable fashion, which aims to reduce the environmental impact of clothing production. This movement emphasizes the use of eco-friendly materials and ethical labor practices. Despite its increasing popularity, sustainable fashion often faces criticism for being more expensive than conventional clothing. Nonetheless, advocates argue that the long-term benefits, such as reduced environmental harm and better working conditions, justify the higher costs. 

Which choice most logically completes the text? 

  1. A) Sustainable fashion is likely to become more affordable as it becomes more mainstream.
  2. B) Critics of sustainable fashion believe that the costs outweigh the environmental benefits.
  3. C) Advocates of sustainable fashion suggest that its higher costs are justified by its long-term benefits.
  4. D) Many people believe that sustainable fashion is a passing trend.
  1. Text 3

Research on sleep patterns shows that people who work night shifts often experience disruptions in their circadian rhythms. This disruption can lead to various health issues, including fatigue and difficulty concentrating. Although some employers have introduced measures to help night shift workers adapt, such as providing more frequent breaks, these measures are often insufficient. As a result, many night shift workers continue to struggle with health problems. 

Which choice best reflects an inference that can be drawn from the text? 

  1. A) Employers have fully resolved the health issues associated with night shifts.
  2. B) Night shift workers often experience health problems despite some improvements in working conditions.
  3. C) Circadian rhythm disruptions are only a minor issue for night shift workers.
  4. D) Frequent breaks are an effective solution to the health problems caused by night shifts.
  1. Text 4

The rise of remote work has led to an increased emphasis on digital communication tools. While these tools offer convenience and flexibility, they can also lead to a sense of isolation among remote workers. Many people who work remotely report feeling disconnected from their colleagues and miss the social interactions of an office environment. Therefore, organizations need to find ways to maintain team cohesion despite physical distance. 

Which choice most logically completes the text? 

  1. A) Remote work is likely to become less popular as more people seek office-based jobs.
  2. B) The use of digital communication tools is sufficient to address the isolation felt by remote workers.
  3. C) Remote workers often feel isolated due to the lack of social interactions with colleagues.
  4. D) Organizations should reduce the reliance on digital tools to improve remote workers’ social experiences.
  1. Text 5

A new study on public transportation usage found that cities with extensive transit networks tend to have higher rates of ridership. The study also showed that improved access to public transportation leads to decreased reliance on personal vehicles, which can reduce traffic congestion and pollution. Despite these benefits, some cities have been slow to invest in expanding their transit systems. This reluctance often stems from budget constraints and resistance to change. 

Which choice best reflects an inference that can be drawn from the text? 

  1. A) Cities with better public transportation networks have lower traffic congestion and pollution.
  2. B) Expanding transit systems is a priority for most city planners.
  3. C) Budget constraints are the main reason for the slow expansion of transit systems in some cities.
  4. D) Personal vehicle usage is unrelated to the availability of public transportation options.
  1. Text 6

The popularity of online learning has increased significantly in recent years, driven by its flexibility and accessibility. However, many educators have raised concerns about the effectiveness of online instruction compared to traditional classroom settings. Studies have shown that students may struggle with self-discipline and motivation in online courses, which can impact their overall learning experience. Consequently, educators are exploring ways to improve online learning environments. 

Which choice most logically completes the text? 

  1. A) Online learning is generally considered more effective than traditional classroom instruction.
  2. B) Students often face challenges with self-discipline and motivation in online learning environments.
  3. C) Educators have found that online learning is as effective as in-person education.
  4. D) Traditional classroom settings are being replaced by online learning in most educational institutions.
  1. Text 7

The use of renewable energy sources has gained traction as concerns about climate change intensify. Solar and wind power are often highlighted for their potential to reduce greenhouse gas emissions. However, some critics argue that the initial investment required for renewable energy infrastructure can be prohibitively high. Despite this, proponents believe that the long-term environmental benefits make it a worthwhile investment. 

Which choice best reflects an inference that can be drawn from the text? 

  1. A) The initial cost of renewable energy infrastructure is a major barrier to its widespread adoption.
  2. B) Renewable energy sources are not effective in reducing greenhouse gas emissions.
  3. C) Critics of renewable energy believe that the long-term environmental benefits are overstated.
  4. D) The long-term environmental benefits of renewable energy are generally considered insignificant.
  1. Text 8

The rise of digital media has transformed how people consume news. Many individuals now rely on social media platforms for real-time updates and breaking news. However, some experts have raised concerns about the reliability of information shared on these platforms. Misinformation and sensationalism can spread quickly, leading to public confusion and distrust. As a result, there is a growing call for improved fact-checking and media literacy education. 

Which choice most logically completes the text? 

  1. A) Social media platforms are generally considered more reliable sources of news than traditional media.
  2. B) The spread of misinformation on social media has led to increased calls for better fact-checking.
  3. C) Digital media has no impact on the public’s trust in news sources.
  4. D) Fact-checking is deemed unnecessary for news shared on social media platforms.
  1. Text 9

As urban areas expand, there is increasing concern about the preservation of green spaces. Parks and natural areas provide important ecological benefits, such as habitat for wildlife and spaces for recreation. Some city planners argue that urban development should prioritize the integration of green spaces to maintain these benefits. However, others contend that the demand for housing and commercial space often outweighs these considerations. 

Which choice best reflects an inference that can be drawn from the text? 

  1. A) Urban development frequently ignores the need for green spaces in expanding cities.
  2. B) The preservation of green spaces is increasingly valued by city planners in urban development.
  3. C) Green spaces are considered less important than housing and commercial development in urban planning.
  4. D) Urban areas do not experience any loss of ecological benefits as they expand.
  1. Text 10

The introduction of electric vehicles (EVs) has been celebrated as a step towards reducing carbon emissions. Many governments offer incentives to encourage the adoption of EVs, such as tax credits and rebates. However, some critics argue that the production of EV batteries has its own environmental impact, including the extraction of raw materials. This has led to a debate about the overall environmental benefits of EVs versus their potential drawbacks. 

Which choice most logically completes the text? 

  1. A) The environmental impact of EV battery production is considered negligible compared to the benefits of EVs.
  2. B) The debate about EVs highlights concerns about the trade-offs between their environmental benefits and their production impact.
  3. C) EVs are universally accepted as having no environmental drawbacks.
  4. D) Government incentives for EVs are viewed as unnecessary due to the lack of environmental concerns.

Grammar

1, 2, 4, 6, 7 – 3, 5, 8, 9, 10

  1. The ancient Egyptian pyramids, built over 4,000 years ago, ______ a lasting symbol of human achievement.

   Which choice completes the text so that it conforms to the conventions of Standard English? 

   Choose 1 answer:

   – A) remain

   – B) remains

   – C) has remained

   – D) is remaining

  1. The committee of scientists ______ to publish their findings on climate change by the end of the year.

   Which choice completes the text so that it conforms to the conventions of Standard English? 

   Choose 1 answer:

   – A) plans

   – B) plan

   – C) planning

   – D) will have planned

  1. Marie Curie, known for her pioneering research on radioactivity, ______ two Nobel Prizes in her lifetime, making her the first person to achieve this honor.

   Which choice completes the text so that it conforms to the conventions of Standard English? 

   Choose 1 answer:

   – A) awarded

   – B) was awarding

   – C) has awarded

   – D) was awarded

  1. Due to the intense competition in the market, the company ______ its product line to appeal to a broader audience.

   Which choice completes the text so that it conforms to the conventions of Standard English? 

   Choose 1 answer:

   – A) expands

   – B) expanding

   – C) expanded

   – D) to expand

  1. The spacecraft’s mission, which lasted for nearly ten years, ______ valuable data about the outer planets.

   Which choice completes the text so that it conforms to the conventions of Standard English? 

   Choose 1 answer:

   – A) collect

   – B) collecting

   – C) collected

   – D) has collected

  1. In her book, the author provides a detailed account of the lives of the ancient Greeks, emphasizing ______ customs and traditions.

   Which choice completes the text so that it conforms to the conventions of Standard English? 

   Choose 1 answer:

   – A) Greeks’

   – B) Greek’s

   – C) Greeks

   – D) Greek

  1. The organization’s members met to discuss ______ plans for the upcoming year.

   Which choice completes the text so that it conforms to the conventions of Standard English? 

   Choose 1 answer:

   – A) their

   – B) his

   – C) its

   – D) there

  1. The children, who had been playing outside all day, ______ inside when it started to rain.

   Which choice completes the text so that it conforms to the conventions of Standard English? 

   Choose 1 answer:

   – A) run

   – B) running

   – C) runs

   – D) ran

  1. The inventor’s new device, which was unveiled at the technology conference, ______ several innovative features that have never been seen before.

   Which choice completes the text so that it conforms to the conventions of Standard English? 

   Choose 1 answer:

   – A) incorporates

   – B) incorporating

   – C) incorporated

   – D) to incorporate

  1. In the museum exhibit, visitors can see the artist’s original sketches along with ______ final paintings.

    Which choice completes the text so that it conforms to the conventions of Standard English? 

    Choose 1 answer:

    – A) his

    – B) their

    – C) her

    – D) its

11 Ancient Greek theaters were architectural marvels, often built into the slopes of hills. Designed to enhance acoustics and visibility, ______.

Which choice completes the text so that it conforms to the conventions of Standard English?

Choose 1 answer:

  1. A) the performances held in these theaters could be heard clearly from every seat.
  2. B) the sloping hillsides provided a natural amphitheater for the audience.
  3. C) these theaters ensured that all attendees could hear and see the performances clearly.
  4. D) every seat in the theater allowed the audience to enjoy the performances without any obstructions.

Grammar

Practice Set: Clause and Sentence Boundaries with Punctuation

1,4,7

2,5,8

  1. Punctuation (Comma Usage):

   The artist Pablo Picasso, known for his unique style and innovative techniques, was a pioneer in the world of modern art. His painting “Guernica” depicts the horrors of war and is considered one of his masterpieces. This powerful work, ______ created in response to the bombing of the town of Guernica during the Spanish Civil War.

   – A) which was

   – B) which, was

   – C) which: was

   – D) which—was

  1. Clauses (Dependent and Independent Clauses):

   During the Renaissance, scholars from across Europe traveled to Italy to study classical texts and artwork. They believed that these works held the key to understanding human nature and ______ the mysteries of the universe.

   – A) unlocking,

   – B) unlocking;

   – C) unlocking:

   – D) unlocking

  1. Supplements (Dashes):

   The Great Wall of China—stretching over 13,000 miles—has stood for centuries as a testament to the engineering prowess of ancient Chinese civilizations. This colossal structure was originally built to protect against invasions, but it ______ a symbol of China’s enduring strength and resilience.

   – A) also became

   – B) also, became

   – C) also—became

   – D) also became:

  1. Punctuation (Colon Usage):

   Nikola Tesla, a prolific inventor and electrical engineer, made significant contributions to the development of modern electrical systems. His inventions include the Tesla coil, alternating current (AC) electricity, and ______ wireless communication.

   – A) radio: and

   – B) radio; and

   – C) radio, and

   – D) radio—and

  1. Clauses (Sentence Boundaries):

   The Eiffel Tower is one of the most recognizable landmarks in the world. Originally intended as a temporary exhibit for the 1889 World’s Fair in Paris, it has become a permanent symbol of the city and ______ the spirit of France.

   – A) reflects,

   – B) reflects;

   – C) reflects:

   – D) reflects

  1. Supplements (Parentheses):

   The American Revolution (1775-1783) marked a pivotal moment in history when the thirteen American colonies won independence from British rule. This event laid the foundation for the United States, and ______ the principles of liberty and democracy.

   – A) established

   – B) established,

   – C) established:

   – D) established—

  1. Punctuation (Semicolon Usage):

   The Amazon Rainforest is often referred to as the “lungs of the Earth”; it produces 20% of the world’s oxygen. This vast and diverse ecosystem is home to millions of species, many of which are ______ yet to be discovered by scientists.

   – A) yet

   – B) yet:

   – C) yet,

   – D) yet;

  1. Clauses (Elliptical Constructions):

   Albert Einstein, renowned for his theory of relativity, revolutionized the way we understand space and time. His groundbreaking work in physics earned him a Nobel Prize, and his name became synonymous with ______ intelligence.

   – A) scientific

   – B) scientific:

   – C) scientific,

   – D) scientific;

  1. Supplements (Appositives):

   Marie Curie—winner of two Nobel Prizes in different scientific fields—was a trailblazer for women in science. Her research on radioactivity not only advanced the understanding of atomic structure but also ______ the treatment of cancer.

   – A) led to

   – B) led; to

   – C) led: to

   – D) led, to

  1. Punctuation (Dash vs. Comma):

    The Statue of Liberty, a gift from France to the United States, is a symbol of freedom and democracy. The statue was dedicated on October 28, 1886, and ______ stands as an iconic monument in New York Harbor.

    – A) today

    – B) today—

    – C) today,

    – D) today:

Transition

5,8,1 – 2,4,6 – 3,9,10

  1.  

Many people believe that success comes solely from hard work. ______, luck and timing can also play crucial roles in achieving one’s goals. 

  1. For example,
  2. Therefore,
  3. In addition,
  4. However,
  1.  

The construction of the new highway has disrupted local wildlife habitats. ______, environmental groups have called for additional measures to protect endangered species. 

  1. As a result,
  2. For instance,
  3. Nevertheless,
  4. Furthermore,
  1.  

The team prepared thoroughly for the competition, practicing every day for weeks. ______, they still felt nervous on the day of the event. 

  1. In contrast,
  2. Consequently,
  3. Nonetheless,
  4. For example,
  1.  

Many classic novels have been adapted into movies. ______, “Pride and Prejudice” has had several film versions, each with its own interpretation of the characters. 

  1. On the other hand,
  2. In other words,
  3. Likewise,
  4. For instance,
  1.  

The museum recently opened a new exhibit featuring ancient artifacts. ______, it has seen a significant increase in visitor numbers. 

  1. Nevertheless,
  2. As a result,
  3. For example,
  4. Similarly,
  1.  

The company’s profits have been steadily declining over the past year. ______, management is considering restructuring its operations to cut costs. 

  1. Similarly,
  2. In contrast,
  3. As a result,
  4. Moreover,
  1.  

The chef carefully prepared the dish, using only the freshest ingredients. ______, the presentation was flawless, with each element arranged meticulously on the plate. 

  1. For instance,
  2. Meanwhile,
  3. However,
  4. Furthermore,
  1.  

Physical exercise is essential for maintaining a healthy body. ______, it also benefits mental health by reducing stress and improving mood. 

  1. In fact,
  2. On the other hand,
  3. Therefore,
  4. Similarly,
  1.  

The artist’s early works were characterized by bold colors and abstract forms. ______, his later pieces focused more on realism and detailed depictions of nature. 

  1. However,
  2. In addition,
  3. Likewise,
  4. As a result,
  1.  

The conference provided an opportunity for professionals to network and share ideas. ______, several new partnerships were formed that could lead to future collaborations. 

  1. In contrast,
  2. For example,
  3. As a result,
  4. Nevertheless,

From the Notes

1,2,4 – 5,6,8

  1. While researching a topic, a student has taken the following notes:

   – The Great Wall of China is a series of fortifications made of various materials, including earth, wood, and stone.

   – The wall was built primarily to protect against invasions from northern tribes.

   – Construction of the wall began as early as the 7th century BCE and continued until the 16th century CE.

   – The wall stretches over 13,000 miles across northern China.

   – The student wants to highlight the main purpose of the Great Wall of China. Which choice most effectively uses relevant information from the notes to accomplish this goal?

   Choose 1 answer:

  1. The Great Wall of China stretches over 13,000 miles and is made of various materials.
  2. The Great Wall of China was built from the 7th century BCE to the 16th century CE to protect against invasions from northern tribes.
  3. The Great Wall of China is a series of fortifications made of earth, wood, and stone.
  4. The Great Wall of China, which is over 13,000 miles long, was constructed to protect against northern invasions.
  1. While researching a topic, a student has taken the following notes:

   – The Amazon rainforest is the largest tropical rainforest in the world.

   – It is home to an incredibly diverse range of flora and fauna.

   – The rainforest plays a critical role in regulating the global climate.

   – It is located in South America and spans across multiple countries.

   – The student wants to emphasize the rainforest’s role in climate regulation. Which choice most effectively uses relevant information from the notes to accomplish this goal?

   Choose 1 answer:

  1. The Amazon rainforest, located in South America, is the largest tropical rainforest in the world.
  2. The Amazon rainforest, spanning multiple countries, is crucial for regulating the global climate.
  3. The Amazon rainforest is home to a wide variety of plants and animals and is the largest tropical rainforest.
  4. The Amazon rainforest plays a critical role in climate regulation and is located in South America.
  1. While researching a topic, a student has taken the following notes:

   – Marie Curie was a pioneering physicist and chemist.

   – She won two Nobel Prizes in different scientific fields.

   – Her research on radioactivity paved the way for advancements in medical treatments.

   – She was the first woman to win a Nobel Prize.

   – The student wants to highlight Marie Curie’s achievements in science. Which choice most effectively uses relevant information from the notes to accomplish this goal?

   Choose 1 answer:

  1. Marie Curie was a physicist and chemist who won two Nobel Prizes.
  2. Marie Curie’s research on radioactivity was groundbreaking and led to advancements in medical treatments.
  3. Marie Curie, the first woman to win a Nobel Prize, made significant contributions to the field of radioactivity.
  4. Marie Curie, a pioneering scientist, won two Nobel Prizes and was the first woman to achieve this honor.
  1. While researching a topic, a student has taken the following notes:

   – The internet was developed in the late 20th century.

   – It has drastically changed the way people communicate and access information.

   – The development of the internet involved several key technological advancements.

   – It has had a profound impact on various aspects of modern life.

   – The student wants to focus on how the internet has changed communication. Which choice most effectively uses relevant information from the notes to accomplish this goal?

   Choose 1 answer:

  1. The internet, developed in the late 20th century, has changed the way people communicate.
  2. The internet has had a profound impact on modern life and involved several key technological advancements.
  3. The development of the internet involved key technological advancements and has drastically changed communication.
  4. The internet has changed communication and access to information in profound ways.
  1. While researching a topic, a student has taken the following notes:

   – The Renaissance was a cultural movement that began in Italy in the 14th century.

   – It marked the transition from the medieval period to the early modern age.

   – The movement emphasized the revival of classical learning and wisdom.

   – Major figures of the Renaissance include Leonardo da Vinci and Michelangelo.

   – The student wants to highlight the Renaissance’s impact on learning and wisdom. Which choice most effectively uses relevant information from the notes to accomplish this goal?

   Choose 1 answer:

  1. The Renaissance, which began in Italy in the 14th century, emphasized classical learning and wisdom.
  2. The Renaissance was a cultural movement that marked the transition from the medieval period to the early modern age.
  3. Major figures of the Renaissance include Leonardo da Vinci and Michelangelo, who contributed to the revival of classical learning.
  4. The Renaissance, a cultural movement starting in Italy, impacted learning and wisdom by reviving classical ideas.
  1. While researching a topic, a student has taken the following notes:

   – The human body has several major systems, including the circulatory, respiratory, and digestive systems.

   – Each system has specific functions that contribute to overall health and well-being.

   – The circulatory system transports blood and nutrients throughout the body.

   – The respiratory system is responsible for breathing and gas exchange.

   – The student wants to emphasize the specific functions of the major systems. Which choice most effectively uses relevant information from the notes to accomplish this goal?

   Choose 1 answer:

  1. The human body has major systems, such as the circulatory and respiratory systems, each with specific functions.
  2. The circulatory system transports blood and nutrients, while the respiratory system handles breathing and gas exchange.
  3. The digestive system contributes to overall health, alongside the circulatory and respiratory systems.
  4. Major systems in the human body, including the circulatory, respiratory, and digestive systems, each have specific functions.
  1. While researching a topic, a student has taken the following notes:

   – Mount Everest is the highest peak in the world.

   – It is located in the Himalayas on the border between Nepal and Tibet.

   – The mountain stands at approximately 8,848 meters above sea level.

   – Climbing Mount Everest is considered a significant challenge due to its extreme altitude.

   – The student wants to highlight the challenge of climbing Mount Everest. Which choice most effectively uses relevant information from the notes to accomplish this goal?

   Choose 1 answer:

  1. Mount Everest, located in the Himalayas, is the highest peak in the world and a major climbing challenge.
  2. Climbing Mount Everest, the highest peak in the world, is considered a significant challenge due to its altitude.
  3. Mount Everest stands at approximately 8,848 meters above sea level and is a notable challenge for climbers.
  4. The Himalayas, where Mount Everest is located, features the world’s highest peak, making climbing a significant challenge.
  1. While researching a topic, a student has taken the following notes:

   – The Great Barrier Reef is the world’s largest coral reef system.

   – It is located off the coast of Queensland, Australia.

   – The reef is home to a diverse range of marine life.

   – Coral reefs play an important role in ocean ecosystems.

   – The student wants to emphasize the reef’s significance in marine ecosystems. Which choice most effectively uses relevant information from the notes to accomplish this goal?

   Choose 1 answer:

  1. The Great Barrier Reef is home to a diverse range of marine life and plays a crucial role in ocean ecosystems.
  2. Located off the coast of Queensland, Australia, the Great Barrier Reef is the world’s largest coral reef system.
  3. The Great Barrier Reef, the largest coral reef system, supports various marine species and contributes to ocean ecosystems.
  4. Coral reefs, including the Great Barrier Reef, play an important role in marine ecosystems.
  1. While researching a topic, a student has taken the following notes:

   – The Eiffel Tower is a wrought-iron lattice tower located in Paris, France.

   – It was constructed between 1887 and 1889 as the entrance arch for the 1889 World’s Fair.

   – The tower stands approximately 324 meters tall.

   – It is one of the most recognizable structures in the world.

   – The student wants to highlight the historical purpose of the Eiffel Tower. Which choice most effectively uses relevant information from the notes to accomplish this goal?

   Choose 1 answer:

  1. The Eiffel Tower, constructed between 1887 and 1889, was built as the entrance arch for the 1889 World’s Fair.
  2. The Eiffel Tower, located in Paris, is a famous landmark and stands 324 meters tall.
  3. Built as the entrance arch for the 1889 World’s Fair, the Eiffel Tower is one of the most recognizable structures in the world.
  4. The Eiffel Tower, standing 324 meters tall, was constructed as part of the 1889 World’s Fair in Paris.
  1. While researching a topic, a student has taken the following notes:

   – The moon is Earth’s only natural satellite.

   – It orbits Earth at an average distance of about 384,400 kilometers.

   – The moon’s gravitational pull affects Earth’s tides.

   – It has a significant impact on various aspects of life on Earth.

   – The student wants to emphasize the moon’s influence on Earth’s tides. Which choice most effectively uses relevant information from the notes to accomplish this goal?

   Choose 1 answer:

  1. The moon’s gravitational pull affects Earth’s tides and has a significant impact on life on Earth.
  2. Orbiting Earth at an average distance of about 384,400 kilometers, the moon is Earth’s only natural satellite.
  3. The moon’s impact on tides is significant, and it influences various aspects of life on Earth.
  4. The moon, Earth’s only natural satellite, orbits at about 384,400 kilometers and affects tides

Information and Ideas

Textual Evidence

### Passage 1

Dr. Emily Cheng and her team conducted a study on the effects of urbanization on bird species diversity in metropolitan areas. They found that while some species thrive in urban environments, others decline or disappear entirely. The team concluded that the key factor in whether a species thrives or declines is its ability to adapt to the rapid changes in habitat brought about by urbanization, such as the loss of green spaces and the increase in pollution levels.

**Which finding, if true, would most directly support Dr. Cheng’s conclusion about the impact of urbanization on bird species?**

– (A) Bird species that thrive in urban areas tend to have diets that are more varied than those that decline.
– (B) Species that decline in urban environments are often those that require large, undisturbed territories to breed successfully.
– (C) The number of bird species that thrive in urban areas is greater than the number that decline.
– (D) Pollution levels in metropolitan areas have a greater impact on bird species than the availability of green spaces.

### Passage 2

A recent study by sociologist Dr. Marissa Ford examined the effects of social media use on teenagers’ self-esteem. The study found that teenagers who spend more than three hours a day on social media are more likely to report feelings of inadequacy and low self-worth compared to those who spend less time online. Dr. Ford hypothesized that this effect is due to the constant exposure to idealized images and lifestyles that are prevalent on social media platforms.

**Which of the following findings would most strongly support Dr. Ford’s hypothesis?**

– (A) Teenagers who are active on social media report higher levels of stress and anxiety than those who are not.
– (B) The majority of teenagers who spend more than three hours a day on social media also report higher levels of peer pressure.
– (C) Teenagers who follow celebrities and influencers on social media are more likely to compare themselves negatively to others.
– (D) The time teenagers spend on social media is often focused on engaging with friends rather than consuming content.

### Passage 3

Economist Dr. Kevin Yates analyzed the economic impact of the growing trend of remote work. He found that while remote work offers significant cost savings for companies, it also presents challenges such as reduced collaboration and innovation. Dr. Yates concluded that companies must find a balance between allowing remote work and maintaining an environment that fosters creativity and teamwork.

**Which of the following statements, if true, would most weaken Dr. Yates’ conclusion?**

– (A) Companies that allow remote work report higher employee satisfaction and retention rates than those that do not.
– (B) Innovation and collaboration have been shown to thrive in virtual work environments with proper tools and management.
– (C) The cost savings from remote work can be reinvested in technologies that enhance virtual collaboration.
– (D) Employees working remotely are more likely to report feelings of isolation and decreased motivation.

### Passage 4

Dr. Rachel Monroe conducted a study on the effectiveness of mindfulness meditation in reducing stress among healthcare workers. Her research showed that participants who practiced mindfulness meditation for 20 minutes a day over an eight-week period reported significantly lower stress levels than those who did not engage in any form of meditation. Dr. Monroe suggests that mindfulness meditation could be a valuable tool in addressing burnout in the healthcare profession.

**Which of the following findings would best support Dr. Monroe’s suggestion?**

– (A) Healthcare workers who practice mindfulness meditation regularly report fewer sick days than those who do not.
– (B) The reduction in stress levels was more pronounced in healthcare workers who practiced mindfulness meditation for longer periods.
– (C) Healthcare workers who practiced mindfulness meditation were more likely to report improved focus and concentration at work.
– (D) Participants who practiced mindfulness meditation also reported improved sleep quality.

### Passage 5

In her research on ancient civilizations, archaeologist Dr. Laura Bennett discovered that the collapse of the Indus Valley Civilization may have been accelerated by a series of severe droughts. By analyzing sediment layers, she found evidence of prolonged dry periods that coincided with the decline of the civilization. Dr. Bennett concluded that the civilization’s dependence on a stable climate for agriculture made it particularly vulnerable to environmental changes.

**Which piece of evidence would most strongly support Dr. Bennett’s conclusion?**

– (A) Other ancient civilizations that experienced droughts did not collapse as rapidly as the Indus Valley Civilization.
– (B) The agricultural practices of the Indus Valley Civilization were heavily reliant on river systems that were disrupted by droughts.
– (C) Archaeological evidence shows that the population of the Indus Valley Civilization was decreasing even before the droughts began.
– (D) The Indus Valley Civilization had developed advanced techniques for water storage and irrigation.

### Passage 6

Psychologist Dr. James O’Hara studied the effects of sleep deprivation on decision-making in high-pressure situations. He found that individuals who were sleep-deprived made riskier decisions compared to those who were well-rested. Dr. O’Hara theorized that lack of sleep impairs the brain’s ability to evaluate potential outcomes accurately, leading to a greater likelihood of taking risks.

**Which finding, if true, would most directly support Dr. O’Hara’s theory?**

– (A) Sleep-deprived individuals are more likely to underestimate the negative consequences of their decisions.
– (B) People who are sleep-deprived often experience heightened emotions that influence their decision-making.
– (C) Individuals who get regular sleep tend to be more cautious in high-pressure situations.
– (D) The ability to make decisions quickly is not significantly impaired by sleep deprivation.

### Passage 7

Historian Dr. John Carrington examined the causes of the French Revolution, focusing on the role of economic inequality. He argued that the widening gap between the wealthy and the poor was a critical factor that led to widespread unrest. Dr. Carrington noted that the economic policies of the monarchy exacerbated this inequality, ultimately fueling the revolution.

**Which of the following statements, if true, would most weaken Dr. Carrington’s argument?**

– (A) The French Revolution was also driven by political factors, such as the demand for greater representation and individual rights.
– (B) Other European countries with similar levels of economic inequality did not experience revolutions during the same period.
– (C) The economic policies of the monarchy were designed to benefit the wealthy at the expense of the poor.
– (D) The French Revolution led to the establishment of policies that further entrenched economic inequality.

### Passage 8

A recent study by environmental scientist Dr. Paula Green found that plastic pollution in the oceans is causing significant harm to marine life. Dr. Green’s research focused on the ingestion of microplastics by fish and other sea creatures, leading to health issues and disruptions in the food chain. She suggested that reducing plastic waste is crucial to protecting marine ecosystems.

**Which of the following findings would most strongly support Dr. Green’s suggestion?**

– (A) Marine species that consume microplastics have shown a decrease in reproductive rates.
– (B) The majority of plastic waste in the oceans comes from land-based sources.
– (C) Fish that ingest microplastics often have shorter lifespans and are more susceptible to disease.
– (D) Efforts to reduce plastic waste have been met with resistance due to the convenience of plastic products.

### Passage 9

Linguist Dr. Susan Adler studied language acquisition in bilingual children. She found that children who are exposed to two languages from an early age tend to develop stronger cognitive abilities than those who speak only one language. Dr. Adler theorized that the mental flexibility required to switch between languages enhances overall brain function.

**Which of the following findings would most directly support Dr. Adler’s theory?**

– (A) Bilingual children outperform monolingual children on tasks that require problem-solving and creative thinking.
– (B) The benefits of bilingualism extend to adults who learn a second language later in life.
– (C) Bilingual children are more likely to maintain their language skills into adulthood.
– (D) The cognitive benefits of bilingualism are most pronounced in children who are equally proficient in both languages.

### Passage 10

In her study on the health benefits of regular exercise, Dr. Linda Matthews found that individuals who engage in physical activity for at least 30 minutes a day are less likely to develop chronic illnesses such as heart disease and diabetes. She also discovered that the type of exercise does not significantly affect the health benefits, suggesting that any form of physical activity can be beneficial.

**Which of the following statements, if true, would most weaken Dr. Matthews’ conclusion?**

– (A) Some forms of exercise, such as high-intensity interval training, have been shown to provide greater health benefits than others.
– (B) Individuals who engage in physical activity are also more likely to follow a healthy diet, which contributes to the prevention of chronic illnesses.
– (C) The duration of physical activity is a more important factor in preventing chronic illnesses than the type of exercise.
– (D) Regular exercise can reduce the risk of chronic illnesses, but it is not sufficient to prevent them without other lifestyle changes.

Answer Key with Explanations

  1. B This choice directly supports the conclusion by explaining how species that decline in urban areas are those needing large territories, which aligns with the conclusion about adaptation to urbanization challenges.
  1. C This choice supports the hypothesis by highlighting that teenagers who follow celebrities and influencers are more likely to negatively compare themselves, which aligns with the idea of idealized images affecting self-esteem.
  1. D This choice weakens the conclusion by suggesting that despite challenges, remote work can still be effective with proper tools, contrary to the idea that reduced collaboration and innovation are major issues.
  1.  A This choice supports Dr. Monroe’s suggestion by showing that mindfulness meditation leads to fewer sick days, implying it is effective in reducing stress and addressing burnout.
  1. B This choice supports the conclusion by showing that the Indus Valley Civilization’s dependence on river systems, which were disrupted by droughts, aligns with the impact of environmental changes on their collapse.
  1. A This choice supports Dr. O’Hara’s theory by indicating that sleep-deprived individuals are more likely to underestimate negative outcomes, which supports the theory that sleep deprivation impairs decision-making.

7. D This choice weakens Dr. Carrington’s argument by suggesting that the French Revolution led to policies that further entrenched economic inequality. If the revolution resulted in increased inequality, it challenges the idea that economic inequality was the key driver of the unrest, as the outcome would suggest that inequality was either not resolved or was exacerbated by the revolution.

  1. C This choice supports Dr. Green’s suggestion by showing that fish ingesting microplastics have shorter lifespans and more diseases, emphasizing the harm of plastic pollution.

 

9 A This choice supports Dr. Adler’s theory by showing that bilingual children perform better in cognitive tasks, aligning with the cognitive benefits of bilingualism.

  1. C  This choice weakens Dr. Matthews’ conclusion by suggesting that the duration of physical activity is a more important factor in preventing chronic illnesses than the type of exercise, challenging the idea that any form of exercise is equally beneficial.

Central Ideas and Details

**1.**
*Passage:*
The history of urban planning in New York City is marked by a consistent tension between the desires of real estate developers and the needs of the city’s residents. This dynamic is most evident in the construction of Central Park, which was originally conceived as a recreational area for the city’s elite but eventually became a public space accessible to all. The park’s development, spearheaded by Frederick Law Olmsted, required the displacement of several communities, including Seneca Village, a predominantly Black settlement. Today, Central Park is celebrated as a green oasis in a bustling metropolis, but its origins are a reminder of the sacrifices that were made to bring it into existence.

*Question:*
Which choice best states the main idea of the passage?
a) The construction of Central Park is an example of how urban development can result in the displacement of marginalized communities.
b) Central Park was initially intended for the elite but became a public space accessible to everyone.
c) Frederick Law Olmsted’s vision for Central Park was to create a green oasis in New York City.
d) The history of Central Park highlights the tension between developers and residents in New York City.

**2.**
*Passage:*
In her groundbreaking research on sleep patterns, Dr. Rebecca Clayton discovered that individuals who consistently get less than six hours of sleep per night are more likely to suffer from chronic illnesses such as diabetes and heart disease. Clayton’s work challenges the popular belief that people can function optimally on minimal sleep, suggesting instead that a lack of sleep has profound long-term health consequences. Her findings have prompted renewed discussions on the importance of sleep hygiene and the need for public health campaigns that promote sufficient rest as a critical component of overall wellness.

*Question:*
Which choice best states the main idea of the passage?
a) Dr. Rebecca Clayton’s research highlights the dangers of chronic sleep deprivation.
b) Sleep hygiene is an essential aspect of overall wellness.
c) Public health campaigns should emphasize the importance of sufficient sleep.
d) Individuals need more than six hours of sleep to avoid chronic illnesses.

**3.**
*Passage:*
The rise of e-commerce has transformed the retail landscape, leading to the decline of brick-and-mortar stores across the globe. Consumers now prioritize convenience and variety, often preferring the vast selection offered by online retailers over the more limited options available in physical stores. However, this shift has also raised concerns about the environmental impact of increased packaging waste and the carbon footprint associated with global shipping. As the world moves further into the digital age, the challenge will be to find a balance between the benefits of e-commerce and the need for sustainable practices.

*Question:*
Which choice best states the main idea of the passage?
a) The rise of e-commerce has led to the decline of traditional retail stores.
b) Online shopping offers consumers more variety and convenience than physical stores.
c) E-commerce’s environmental impact is a growing concern in the digital age.
d) The transformation of retail has prompted a need for sustainable practices in e-commerce.

**4.**
*Passage:*
The practice of social distancing during the COVID-19 pandemic highlighted the importance of digital communication tools in maintaining social connections. Platforms such as Zoom, Skype, and WhatsApp became lifelines for people isolated from their friends and families. While these tools allowed for continued interaction, they also exposed the limitations of virtual communication, such as the lack of physical presence and the potential for misunderstandings. As the world slowly returns to normal, the lessons learned from this period may lead to a more nuanced understanding of how technology can support, but not replace, in-person interactions.

*Question:*
Which choice best states the main idea of the passage?
a) Social distancing emphasized the importance of digital communication tools during the COVID-19 pandemic.
b) Virtual communication platforms became essential for maintaining social connections during the pandemic.
c) The limitations of virtual communication were exposed during the pandemic.
d) The pandemic may lead to a better understanding of the role of technology in human interactions.

**5.**
*Passage:*
In recent years, there has been a growing recognition of the importance of mental health in the workplace. Employers are increasingly offering wellness programs that include mental health support, such as counseling services and stress management workshops. This shift reflects a broader understanding that mental well-being is crucial to employee productivity and overall job satisfaction. However, challenges remain in reducing the stigma associated with mental health issues, which can prevent employees from seeking the help they need.

*Question:*
Which choice best states the main idea of the passage?
a) Mental health support in the workplace is becoming more common.
b) Employers are recognizing the link between mental health and productivity.
c) The stigma around mental health issues remains a significant challenge in the workplace.
d) Wellness programs are helping to address mental health in the workplace.

**6.**
*Passage:*
The development of renewable energy technologies, such as solar and wind power, has been hailed as a crucial step towards combating climate change. These technologies offer the potential to reduce reliance on fossil fuels and lower greenhouse gas emissions. However, the transition to renewable energy sources is not without challenges, including the need for significant infrastructure investment and the intermittent nature of energy generation. As the world moves towards a more sustainable future, policymakers must navigate these obstacles to ensure that the benefits of renewable energy can be fully realized.

*Question:*
Which choice best states the main idea of the passage?
a) Renewable energy technologies are essential for combating climate change.
b) The transition to renewable energy requires significant infrastructure investment.
c) Policymakers must address the challenges of renewable energy to realize its benefits.
d) Solar and wind power have the potential to reduce reliance on fossil fuels.

**7.**
*Passage:*
Advancements in artificial intelligence (AI) have led to significant improvements in various fields, from healthcare to finance. AI algorithms can analyze vast amounts of data more quickly and accurately than humans, enabling new insights and efficiencies. However, these advancements also raise ethical questions about the role of AI in decision-making processes, particularly when it comes to issues of privacy, bias, and accountability. As AI continues to evolve, society must grapple with the implications of these technologies to ensure they are used responsibly.

*Question:*
Which choice best states the main idea of the passage?
a) AI has led to significant improvements in various fields.
b) The use of AI in decision-making raises important ethical questions.
c) AI algorithms can analyze data more quickly and accurately than humans.
d) Society must ensure that AI technologies are used responsibly.

**8.**
*Passage:*
The exploration of Mars has long been a goal for space agencies around the world. Recent advancements in technology have brought humanity closer to the possibility of manned missions to the Red Planet. However, the journey to Mars presents numerous challenges, including the need for life support systems, radiation protection, and the psychological effects of long-duration space travel. Despite these obstacles, the potential for scientific discovery and the possibility of establishing a human presence on another planet continue to drive efforts to reach Mars.

*Question:*
Which choice best states the main idea of the passage?
a) The exploration of Mars has been a long-standing goal for space agencies.
b) Recent technological advancements have made manned missions to Mars more feasible.
c) The journey to Mars presents significant challenges that must be overcome.
d) The potential for discovery and human presence on Mars drives exploration efforts.

**9.**
*Passage:*
The global spread of invasive species is a growing environmental concern. Species that are introduced to new ecosystems, either intentionally or accidentally, can disrupt local habitats, outcompete native species, and cause significant ecological and economic damage. Efforts to control invasive species often involve a combination of prevention, early detection, and rapid response. However, the complexity of ecosystems and the adaptability of invasive species make these efforts challenging. As globalization continues, the need for coordinated international action to address invasive species becomes increasingly urgent.

*Question:*
Which choice best states the main idea of the passage?
a) Invasive species pose a significant threat to global ecosystems.
b) Efforts to control invasive species are hindered by the complexity of ecosystems.
c) Globalization has increased the spread of invasive species.
d) Coordinated international action is needed to address the issue of invasive species.

**10.**
*Passage:*
The concept of universal basic income (UBI) has gained attention as a potential solution to economic inequality and job displacement caused by automation. Proponents argue that providing a guaranteed income to all citizens would ensure financial security and allow people to pursue education, entrepreneurship, or other activities without the fear of poverty. Critics, however, raise concerns about the cost of implementing UBI and its potential impact on work incentives. As automation continues to reshape the job market, the debate over UBI is likely to intensify.

*Question:*
Which choice best states the main idea of the passage?
a) Universal basic income is proposed as a solution to economic inequality and job displacement.
b) Proponents of UBI argue that it would provide financial security for all citizens.
c) The debate over UBI is likely to intensify as automation affects the job market.
d) Critics raise concerns about the cost and impact of implementing UBI.

Answer Key with Explanations

**1.** **Answer: A**
*Explanation:* The passage emphasizes the displacement of marginalized communities, particularly Seneca Village, during the construction of Central Park. This makes option A, which highlights urban development and its impact on marginalized communities, the best choice.

**2.** **Answer: A**
*Explanation:* The passage discusses Dr. Rebecca Clayton’s findings on the dangers of chronic sleep deprivation, particularly the link between lack of sleep and chronic illnesses. Option A accurately captures this main idea.

**3.** **Answer: C**
*Explanation:* The passage touches on the environmental concerns related to the rise of e-commerce, especially the increased packaging waste and carbon footprint. Option C, which addresses the need for sustainable practices in e-commerce, best reflects the passage’s main idea.

**4.** **Answer: D**
*Explanation:* The passage discusses the importance of digital communication tools during the pandemic but also mentions the limitations of these tools. Option D best encapsulates the idea that while technology supports human interactions, it cannot replace in-person interactions.

**5.** **Answer: B**
*Explanation:* The passage focuses on how employers are increasingly recognizing the link between mental health and employee productivity. Option B, which highlights this connection, best represents the passage’s main idea.

**6.** **Answer: C**
*Explanation:* The passage acknowledges the importance of renewable energy while also discussing the challenges associated with transitioning to these technologies. Option C, which emphasizes the need for policymakers to address these challenges, accurately reflects the passage’s main idea.

**7.** **Answer: B**
*Explanation:* The passage explores the ethical questions raised by the use of AI in decision-making, particularly regarding privacy, bias, and accountability. Option B, which addresses these ethical concerns, best captures the passage’s main idea.

**8.** **Answer: C**
*Explanation:* The passage discusses the challenges of a manned mission to Mars, including life support, radiation protection, and psychological effects. Option C, which mentions the significant challenges that must be overcome, best reflects the passage’s main idea.

**9.** **Answer: D**
*Explanation:* The passage highlights the global spread of invasive species and the need for coordinated international action to address this issue. Option D, which emphasizes this need for international action, best represents the main idea of the passage.

**10.** **Answer: C**
*Explanation:* The passage focuses on the debate over universal basic income (UBI) and how it might intensify as automation continues to affect the job market. Option C, which mentions the likelihood of this debate intensifying, best reflects the main idea of the passage.

Inference

Question 1: During the Bourbon Restoration in France (1814–1830), the right to vote required in part that a person paid at least 300 francs in direct taxes to the government. The four most common taxes (the quatre vieilles) were levied on real estate (both land and buildings); the doors and windows in taxpayer homes; the rental values of homes; and the businesses of artisans and merchants. (Foreign investments were either exempt from taxation or taxed lightly.) Although relatively few people paid the tax on real estate, it was the main means of voter qualification and accounted for over two-thirds of government receipts during this period, suggesting that during the Bourbon Restoration ______

Which choice most logically completes the text? Choose 1 answer:

  • (A) the number of doors and windows in French residences was kept to a minimum but increased after 1830.
  • (B) the voting habits of French artisans and merchants were effective in reducing tax burdens on businesses.
  • (C) those people who had the right to vote most likely had substantial holdings of French real estate.
  • (D) French people with significant foreign investments were unlikely to have the right to vote.

Question 2: The Analects is the most influential collection of sayings and teachings attributed to Confucius, the founder of Confucianism. Compiled by his disciples over the centuries following his death in 479 BCE, the Analects shows remarkable internal consistency and coherence in its themes and principles, such as filial piety, ritual, humaneness, and loyalty. Moreover, many of the anecdotes in the Analects match those found in other historical texts that record the words and deeds of Confucius. However, some passages refer to events and ideas that emerged much later in Chinese history, such as the unification of the empire by Qin Shi Huang in 221 BCE, and reveal the influence of other philosophical schools, such as Legalism and Daoism, which arose after Confucius’s time. Therefore, scholars have concluded that while its core reflects the original teachings of Confucius, the Analects ______

Which choice most logically completes the text? Choose 1 answer:

  • (A) also reflects the changing historical and intellectual circumstances of its compilers.
  • (B) was probably not compiled by Confucius’s disciples, but rather by scholars of Legalism and Daoism.
  • (C) was likely completed before the unification of the empire by Qin Shi Huang in 221 BCE.
  • (D) was not consistent with other historical texts that recorded Confucius’s sayings and deeds.

Question 3: German theater practitioner Bertolt Brecht (1898–1956) believed that theater should elicit an intellectual rather than an emotional response from audiences, provoking them to consider social and political realities that extend beyond the characters and events depicted onstage. Brecht’s influence can be seen in English playwright Caryl Churchill’s 1979 play Cloud 9: although the play sometimes invites empathetic reactions, it primarily works to engage audiences in an interrogation of patriarchy and colonialism, which it does by placing audiences at a distance, thereby encouraging them to ______

Which choice most logically completes the text? Choose 1 answer:

  • (A) reflect on social and political phenomena not directly related to patriarchy and colonialism.
  • (B) focus on the characters’ beliefs about social and political issues as revealed by the characters’ actions.
  • (C) be dispassionate as they think critically about the social and political questions raised by the play.
  • (D) recognize pertinent social and political parallels between Germany during Brecht’s time and England at the time when Churchill was writing Cloud 9.

Question 4: To better understand the burrowing habits of Alpheus bellulus (the tiger pistol shrimp), some studies have used resin casting to obtain precise measurements of the shrimps’ burrows. Resin casting involves completely filling an empty burrow with a liquid plastic that hardens to create a three-dimensional model; however, recovering the model inevitably requires destroying the burrow. In their 2022 study, Miyu Umehara and colleagues discovered that an x-ray computed tomography (CT) scanner can accurately record a burrow’s measurements both at a moment in time and throughout the entire burrow-building process, something that’s impossible with resin casting because ______

Which choice most logically completes the text? Choose 1 answer:

  • (A) it can only be used on burrows below a certain size.
  • (B) it does not allow for multiple castings of the same burrow over time.
  • (C) the process of recovering the model distorts the resin’s shape.
  • (D) the casting process takes more time than A. bellulus takes to construct a burrow.

Question 5: In her 2019 book The Global Republic, historian Franka Raubenheimer argues that the period between 1789 and 1914 saw the emergence of a global order marked by increased interconnectivity and the spread of Western political ideologies. This period was also characterized by the rise of nation-states, as empires began to crumble under the weight of nationalist movements. However, Raubenheimer suggests that the transition to a global order was uneven, with some regions embracing Western political ideologies more readily than others. This observation implies that during this period, ______

Which choice most logically completes the text? Choose 1 answer:

  • (A) some regions experienced significant resistance to Western political ideologies.
  • (B) the rise of nation-states was universally accepted as a positive development.
  • (C) the spread of Western political ideologies was more influential than the rise of nation-states.
  • (D) the emergence of a global order was driven by economic rather than political factors.

Question 6: In a study examining the impact of urban noise on bird communication, researchers found that some species adapted by altering the pitch and duration of their songs. For example, the great tit (Parus major) was observed to sing at a higher pitch in noisy urban environments compared to quieter rural areas. The study also noted that these changes in song characteristics were associated with higher levels of stress hormones in the birds, suggesting that ______

Which choice most logically completes the text? Choose 1 answer:

  • (A) urban noise may have long-term negative effects on the health of bird populations.
  • (B) birds that do not adapt their songs to urban noise may be more successful in breeding.
  • (C) the great tit is the only bird species affected by urban noise.
  • (D) stress hormones are the primary factor influencing song characteristics in birds.

Question 7: In a 2021 study on the effects of climate change on agriculture, researchers found that higher temperatures and changes in precipitation patterns were linked to reduced crop yields in many regions. However, the study also found that some crops, such as wheat and barley, experienced increased yields in certain areas due to longer growing seasons and improved water use efficiency. These findings suggest that ______

Which choice most logically completes the text? Choose 1 answer:

  • (A) climate change will universally lead to lower crop yields across the globe.
  • (B) the effects of climate change on agriculture are complex and region-specific.
  • (C) crops like wheat and barley will become the most important global food sources.
  • (D) farmers should focus on growing crops that are resistant to climate change.

Question 8: In his 2017 book Artificial Intelligence and the Future of Humanity, computer scientist Dr. Priya Nair argues that the development of AI technologies presents both unprecedented opportunities and significant ethical challenges. Nair notes that while AI has the potential to revolutionize industries and improve quality of life, it also raises concerns about privacy, job displacement, and the concentration of power in the hands of a few large tech companies. This perspective suggests that ______

Which choice most logically completes the text? Choose 1 answer:

  • (A) AI technologies will primarily benefit large tech companies at the expense of the general population.
  • (B) the ethical challenges associated with AI are likely to outweigh its potential benefits.
  • (C) the responsible development and regulation of AI is crucial to maximizing its benefits.
  • (D) AI technologies will lead to widespread job displacement across all industries.

Question 9: A 2020 report on the effects of deforestation in the Amazon rainforest highlighted the significant loss of biodiversity due to the destruction of habitats. The report also emphasized the importance of the Amazon as a carbon sink, absorbing large amounts of carbon dioxide from the atmosphere. However, ongoing deforestation threatens this crucial function, potentially accelerating global climate change. This information suggests that ______

Which choice most logically completes the text? Choose 1 answer:

  • (A) deforestation in the Amazon may lead to increased biodiversity in other regions.
  • (B) the preservation of the Amazon rainforest is essential for mitigating climate change.
  • (C) the Amazon rainforest’s role as a carbon sink is more important than its biodiversity.
  • (D) deforestation in the Amazon will have little impact on global climate change.

Question 10: A study conducted by linguists in 2022 examined the impact of bilingualism on cognitive development in children. The researchers found that bilingual children outperformed their monolingual peers on tasks requiring executive function, such as problem-solving and memory recall. However, the study also noted that bilingual children faced challenges in maintaining proficiency in both languages, particularly when one language was less commonly spoken in their community. This finding indicates that ______

Which choice most logically completes the text? Choose 1 answer:

  • (A) bilingualism may offer cognitive advantages at the cost of language proficiency.
  • (B) monolingual children are at a disadvantage in terms of cognitive development.
  • (C) maintaining proficiency in multiple languages is not a significant challenge for bilingual children.
  • (D) bilingual children with less commonly spoken languages tend to outperform their peers in all cognitive tasks.

Answer Key with Explanations

  1. (C) Those people who had the right to vote most likely had substantial holdings of French real estate.

   Explanation: The passage indicates that the right to vote was based in part on the payment of taxes on real estate, and this tax accounted for a large portion of government receipts. Therefore, those with the right to vote likely had significant real estate holdings.

 

  1. (A) Also reflects the changing historical and intellectual circumstances of its compilers.

   Explanation: The passage notes that some elements of the Analects refer to events and ideas that emerged after Confucius’s time, implying that the text reflects the changing circumstances of its compilers.

 

  1. (C) Be dispassionate as they think critically about the social and political questions raised by the play.

   Explanation: Brecht’s approach was to encourage audiences to critically engage with social and political issues, and Churchill’s play follows this method by placing audiences at a distance to provoke critical thinking rather than emotional engagement.

 

  1. (C) The process of recovering the model distorts the resin’s shape.

   Explanation: The passage explains that recovering the resin model inevitably destroys the burrow, which is a limitation of resin casting compared to x-ray CT scanning, which does not have this issue.

 

  1. (A) Some regions experienced significant resistance to Western political ideologies.

   Explanation: The passage indicates that the transition to a global order was uneven, with some regions embracing Western ideologies more readily, suggesting that some regions resisted these ideologies.

 

  1. (A) Urban noise may have long-term negative effects on the health of bird populations.

   Explanation: The passage suggests that changes in song characteristics associated with higher stress levels due to urban noise indicate potential long-term negative effects on bird health.

 

  1. (B) The effects of climate change on agriculture are complex and region-specific.

   Explanation: The passage notes that while some crops experienced increased yields in certain areas due to climate change, others saw reduced yields, highlighting the complexity and regional specificity of climate change effects on agriculture.

 

  1. (C) The responsible development and regulation of AI is crucial to maximizing its benefits.

   Explanation: Nair’s perspective on the opportunities and challenges of AI suggests that balancing its development with ethical considerations is important for maximizing its benefits.

 

  1. (B) The preservation of the Amazon rainforest is essential for mitigating climate change.

   Explanation: The report emphasizes the Amazon’s role as a carbon sink and how deforestation threatens this role, suggesting that preserving the rainforest is crucial for mitigating climate change.

 

  1. (A) Bilingualism may offer cognitive advantages at the cost of language proficiency.

    Explanation: The study found bilingual children outperforming peers in cognitive tasks but also noted challenges in maintaining proficiency in both languages, indicating that cognitive advantages may come with language proficiency challenges.

Craft and Structure

Transitions

  1. In recent years, some economists have argued that gross domestic product (GDP) is an inadequate measure of a nation’s wellbeing, as it fails to account for factors such as income inequality and environmental degradation. _____ they have advocated the use of alternative metrics like the Genuine Progress Indicator (GPI), which adjusts GDP by factoring in the costs of crime, pollution, resource depletion, and other negative effects related to economic activity.
  • -A. To that end, 
  • -B. Nonetheless, 
  • -C. In short, 
  • -D. Likewise,
  1. A 2017 study of sign language learners tested the role of iconicity—the similarity of a sign to the thing it represents—in language acquisition. The study found that the greater the iconicity of a sign, the more likely it was to have been learned. ______ the correlation between acquisition and iconicity was lower than that between acquisition and another factor studied: sign frequency.
  • -A. In fact, 
  • -B. In other words, 
  • -C. Granted, 
  • -D. As a result,
  1. The ancient Greek philosopher Plato posited the existence of an abstract realm of “forms,” where the true essence of every concept or object we perceive in the material world exists as a perfect, unchanging paragon. _____, his student Aristotle claimed that the true essences of things are manifest in their real-world characteristics and functionalities, not in abstract ideals, and can thus be understood through direct observation.
  • -A. Conversely, 
  • -B. Altogether, 
  • -C. Accordingly, 
  • -D. To illustrate,
  1. “Tulip mania”—the rapid rise and sudden fall of the price of tulip bulbs in seventeenth-century Amsterdam—is often cited as an example of the perils of rampant market speculation. However, recent research has demonstrated that the episode was neither as frenzied nor as disastrous as has been thought. The popular myth surrounding it, ______ should be regarded with some skepticism.
  • -A. by contrast, 
  • -B. therefore, 
  • -C. for example, 
  • -D. nevertheless,
  1. The discovery of penicillin marked a significant turning point in medical history, revolutionizing the treatment of bacterial infections. _____, the overuse of antibiotics in subsequent decades has led to the rise of antibiotic-resistant bacteria, posing a major threat to public health.
  • -A. For instance, 
  • -B. On the contrary, 
  • -C. Similarly, 
  • -D. Unfortunately,
  1. Renewable energy sources like solar and wind power are becoming increasingly cost-competitive with traditional fossil fuels. _____, these technologies offer the additional benefit of reducing greenhouse gas emissions, which are a major contributor to climate change.
  • -A. In contrast, 
  • -B. Moreover, 
  • -C. Nevertheless, 
  • -D. Consequently,
  1. Many people assume that introverts are naturally shy and prefer to be alone. _____, introversion is more accurately characterized by a preference for less stimulating environments and a tendency to focus inward.
  • -A. Therefore, 
  • -B. For example, 
  • -C. However, 
  • -D. Likewise,
  1. The Wright brothers are often credited with inventing the airplane. ______, many aviation historians emphasize that their success was built on the contributions of earlier pioneers, such as Otto Lilienthal and Samuel Langley, who laid the groundwork for powered flight.
  • -A. Similarly, 
  • -B. Nonetheless, 
  • -C. Consequently, 
  • -D. Indeed,
  1. Climate change is expected to increase the frequency and severity of extreme weather events, such as hurricanes and droughts. _____, coastal communities are investing in infrastructure improvements to protect against rising sea levels and storm surges.
  • -A. For this reason, 
  • -B. In addition, 
  • -C. On the other hand, 
  • -D. Conversely,
  1. Many athletes rely on sports drinks to replenish electrolytes and maintain energy levels during intense physical activity. _____, studies have shown that for most people, water is just as effective for staying hydrated, and it doesn’t contain added sugars or calories.
  • -A. Moreover, 
  • -B. As a result, 
  • -C. However, 
  • -D. Consequently,

Answers and Explanations

  1. Answer: A. To that end, 

Explanation: “To that end” logically connects the idea that GDP is inadequate to the advocacy for alternative metrics like GPI, indicating a purpose or goal.

  1. Answer: C. Granted, 

Explanation: “Granted” introduces a concession, acknowledging the correlation between acquisition and iconicity before contrasting it with the stronger correlation with sign frequency.

  1. Answer: A. Conversely, 

Explanation: “Conversely” contrasts Plato’s abstract theory with Aristotle’s more practical, observation-based approach.

  1. Answer: D. Nevertheless, 

Explanation: “Nevertheless” is used to contrast the popular myth with the recent research, suggesting skepticism despite common belief.

  1. Answer: D. Unfortunately, 

Explanation: “Unfortunately” highlights the negative consequence (antibiotic resistance) that followed the positive discovery of penicillin.

  1. Answer: B. Moreover, 

Explanation: “Moreover” adds additional information, emphasizing that renewable energy sources not only are cost-competitive but also reduce greenhouse gas emissions.

 

  1. Answer: C. However, 

Explanation: “However” contrasts the common misconception about introverts with the more accurate characterization.

  1. Answer: D. Indeed, 

Explanation: “Indeed” reinforces the statement that the Wright brothers’ success was based on earlier contributions, emphasizing this point.

  1. Answer: A. For this reason, 

Explanation: “For this reason” logically explains why coastal communities are investing in infrastructure improvements in response to the threat of climate change.

  1. Answer: C. However, 

Explanation: “However” contrasts the common practice of using sports drinks with the studies showing that water is often just as effective for hydration.